You are on page 1of 7

1/9/2019 JOM proposals

 Mathematics Writings Codings Music

Anzo's Mathematics
My Problem Proposals to JOM
The idea of hosting a Junior Olympiad of Mathematics (JOM) for the junior
students in the IMO camp ignited my thoughts of creating problems for the
junior students. In particular, I proposed my first problem to the Olympiad
after being inspired by IMO 2012, Problem 4: a problem that caused turmoils
both among the students during the contests, and among the leaders and
coordinators during the coordination sessions. Nevertheless, some beauty of
the problem still exists in this IMO problem, which I extracted it and turned it
into a problem that made itself into N6 on the JOM shortlist.

Problem 5 of the same Olympiad also inspired me to create another geometry


problem, also for the first edition of the Junior Olympiad. Unwilling to
succumb to the fact that I didn't have enough time to attempt this problem on
the contest (blame problem 4), I later attempted it again and found an
elegant solution using poles and polars. That led me to the unexpected
discovery of a geometric identity that eventually became problem G2 on the
JOM shortlist.

Below are the 10 problems I proposed for the first three editions of JOM. Have
fun solving!

JOM 2015, A6. Let (an )n≥0 and (bn )n≥0 be two sequences with arbitrary real
values a0 , a1 , b0 , b1 . For n ≥ 1, let an+1 , bn+1 be defined in this way:

bn−1 + bn an−1 + an
an+1 = , bn+1 =
2 2

Prove that for any constant c > 0 there exists a positive integer N s.t. for all
n > N , |an − bn | < c.

Hide Solution

cn +cn+1
Let (cn )n≥0 be a sequence with cn = an − bn , then cn+2 = −
2
. In the
cn+2 +cn+1 cn −cn+1
same way we obtain cn+3 = − = . Now, by triangle inequality
4 4

and Hence,
1 1
|cn+2 | ≤ (|cn | + |cn+1 |) |cn+3 | ≤ (|cn+1 + |cn |).
2 4

. Repeating the step gives


3
|cn+2 | + |cn+3 | ≤ (|cn | + |cn+1 |)
4
n

. We can take so
3 c
|c2n | + |c2n+1 | ≤ ( ) (|c0 | + |c1 |) N = 2 log 3 ( ) + 2
4 |c0 |+|c1 |
4

1
n
3 2

that for all n ≥ N , |cn | ≤ |cn | + |cn+1 | ≤ (


4
) (|c0 | + |c1 |)

c
log 3 ( )

, as desired.
3 |c |+|c |
0 1
< ( ) 4
(|c0 | + |c1 |) = c
4

https://anzoteh96.github.io/jom_proposals.html 1/7
1/9/2019 JOM proposals

JOM 2014, C6. Let n be a positive integer. At the beginning, two frogs, A
and B are at point 0 of a number line. At each second, they jump to the right
and stop after arriving at n, following the constraints below:
(i) Each of them must jump either 1 or 2 step(s) every second.
(ii) Both of them must arrive at n at the same time.
(iii) Frog A must never be behind B during the jump.
Let f (n) be the number of arrangements of the sequence of jumps for the
two frogs. Prove that f (n) ≥ 2
n−1
.

Hide Solution

Let g(n) be the number of arrangements of the sequence of jumps that


follows the constraint of f (n) except frog A arrives at n + 1 and B arrives at
n simultaneously. The strategy is to induct on both f (n) and g(n) to prove

that f (n), g(n) ≥ 2 . Denote to be


n−1
(a1 + a2 + ⋯ + ak , b1 + b2 + ⋯ + bk )

the sequence of jumps by frogs A and B where ai , bi ∈ {1, 2} .

Base case. f (1) = 1 because (1, 1) is the only sequene. f (2) = 2 because we
have (2, 2), (1 + 1, 1 + 1). g(1) = 1 as we have (2, 1) while g(2) = 2 as we
have (2 + 1, 1 + 1) and (1 + 2, 1 + 1).

Inductive step. Now assume that there is an n ≥ 2 such that


f (k), g(k) ≥ 2
k−1
for all k = 1, 2, ⋯ , n. In counting f (n + 1), we know that
the second last position for A and B can be (n, n), (n − 1, n − 1) or
(n, n − 1). This means

f (n + 1) = f (n) + f (n − 1) + g(n − 1) ≥ 2
n−1
In+ 2
n−2
+ 2
n−2
= 2 .
n

counting g(n + 1), the last step can be (1, 1), (2, 1), (1, 2), (2, 2) whereby the
second last position is (n + 1, n), (n, n), (n + 1, n − 1), (n, n − 1). Thus
. Q.E.D.
n−1 n−1 n−2 n
g(n + 1) ≥ g(n) + f (n) + g(n − 1) ≥ 2 + 2 + 2 ≥ 2

Proposer's note: equality for f (n) holds iff n ≤ 4.

JOM 2014, Problem 5 (G5). Given △ABC with circumcircle Γ and


circumcentre O. Let X to be a point on Γ. Let XC1 , XB1 to be feet of
perpendiculars from X to lines AB and AC . Let ωC to be circle with centre
the midpoint of AB and passing through C1 . Define ωB similarly.
Prove that ωB and ωC has a common point on XO.

Hide Solution

Let P be the projection from A to XO and we claim that P lies on ωB and ωC


. Let the midpoint of AC be MB . Since ∠XB1 A = XP A = 90∘ , we have
X, B1 , P , A concyclic and ∠(P B1 , B1 A) = ∠(P X, XA) and

∠(B1 P , P X) = ∠(B1 A, AX). Also, since O, MB , A, P are concyclic (becuse

∠OMB A = ∠OP A = 90 ), we have ∠(OP , P MB ) = ∠(OA, AMB ). So


∠(P B1 , B1 A) = ∠(P B1 , B1 MB ) = ∠(P X, XA) = ∠(OX, XA)

= ∠(XA, AO) = ∠(XA, AB1 ) + ∠(AB1 , AO)

= ∠(XP , P B1 ) + ∠(AMB , AO) = ∠(OP , P B1 ) + ∠(P MB , P O)

= ∠(P MB , P B1 ) . This means M B B1 = M B P and P lies on ωB . Likewise, it


lies on ωC .
https://anzoteh96.github.io/jom_proposals.html 2/7
1/9/2019 JOM proposals

JOM 2014, N11. Prove that for all nonzero integers c there exists composite
positive integers a, b such that:
(i) a − b = c.
(ii) gcd(a, b) = gcd(a, c) = gcd(b, c) = 1.

Hide Solution

Solution 1. We may assume that c > 0 since if a − b = c then b − a = −c.


Let p, q be distinct primes not divisible by c. Consider the set
P = {ap |a ∈ N}. Since p ⊥ q , P contains the complete residues mod q ,
2 2 2 2

and there exist infinitely many a such that ap2 ≡ c (mod q 2 ). So there exists
a, b with ap2 − bq 2 = c , and b positive for a sufficiently large. Clearly, ap
2
and
bq
2
are composite.

Notice that a may not be relatively prime to c. However, for any x ∈ N we


have (a + xq 2 )p2 − (b + xp2 )q 2 = c. Moreover, c, p, q are pairwise relatively
prime. Thus by Dirichlet's theorem, there are infinitely many x such that
a + xq
2
and c are relatively prime. The rest of the conclusion follows from
that gcd(a, b) = gcd(a, c) = gcd(b, c).

Solution 2. W.L.O.G. let c > 0. Let a = (2c + 1)! + 2c + 1 and


b = (2c + 1)! + c + 1. gcd(a, c) = gcd(2c + 1, c) = 1 since c|(2c + 1)!.

Moreover, 2c + 1|(2c + 1)! + 2c + 1 and c + 1|(2c + 1)! + c + 1 and both


2c + 1, c + 1 > 1.

JOM 2013, A5. Find all polynomials P ∈ R[x] such that


P (x) + P (y) + P (z) = 0 ⇒ x + y + z = 0 .

Hide Solution

Answer. All P ∈ R[x] such that P (x) = ax or P (x) = ax2n Q(x) for some
a ≠ 0, n ≥ 0, and Q(x) with no real root.

Solution 1 (Justin Lim). If P (x) = 0 for some x, then 3P (x) = 0 ⇒ x = 0.


Suppose P (0) ≠ 0, then P has no real root, so we're done. Suppose
P (0) = 0. If P takes both positive and negative values, then P is surjective

on R. Take infinitely many pairs x, y such that


P (x) + P (y) = 0 ⇒ x + y = 0 ⇒ P (x) + P (−x) = 0. It follows that

P (x) + P (−x) ≡ 0 for all real x. For fixed x, choose z such that

P (x) + P (x) + P (z) = 0 ⇒ z = −2x ⇒ 2P (x) = P (2x), which gives

deg(P ) = 1.

Suppose P takes only positive values or only negative values. Then clearly P
has even multiplicity at 0, so P (x) = x2n Q(x) for some polynomial Q(x)
without real roots. Conversely, it is easy to check that the above polynomials
work, since for P (x) = ax, P (x) + P (y) + P (z) = a(x + y + z) and
a(x + y + z) = 0 ⇔ x + y + z = 0. For P (x) = x Q(x), clearly
2n

P (x) + P (y) + P (z) ≠ 0 for x, y, z with not all of them equal to 0.

JOM 2013, Problem 4 (C3). Let n be a positive integer. A pseudo-Gangnam


Style is a dance competition between players A and B. At time 0, both

https://anzoteh96.github.io/jom_proposals.html 3/7
1/9/2019 JOM proposals

players face to the north. For every k ≥ 1, at time 2k − 1, player A can either
choose to stay stationary, or turn 90∘ clockwise, and player B is forced to
follow him; at time 2k, player B can either choose to stay stationary, or turn
90 clockwise, and player A is forced to follow him.

After time n, the music stops and the competition is over. If the final position
of both players is north or east, A wins. If the final position of both players is
south or west, B wins. Determine who has a winning strategy when:
(a) n = 20132012 ;
(b) n = 2013 .
2013

Hide Solution

(a) At time 1, let A choose to be stationary. For the subsequent steps (i.e.
k ≥ 1), if B stays stationary at time 2k then A moves 90 at time 2k + 1. On

the other hand, if B turns 90 at time 2k then A stays stationary at time


2k + 1. So their positions form a cycle of period 8. In particular, at time


8k + 1 both players will come to original position, i.e. north. Since

≡ 1 (mod 8), their final position will be north and A will win.
2012
2013

(b) For k ≥ 1, if A stays stationary at time 2k − 1 then B moves 90∘ at time


2k. On the other hand, if A turns 90 at time 2k − 1 then B stays stationary

at time 2k. Again, it is a cycle of period 8, and at time 8k + 4 both players


are facing south. Since 20132013 ≡ 5 (mod 8), we know that at time
− 1 both players are at south, and for the final step A can only
2013
2013
choose to stay facing south or turn 90∘ clockwise to face west. So B will
definitely win.

JOM 2013, G2. Let ω1 and ω2 be two circles, with centres O1 and O2
respectively, intersecting at X and Y . Let a line tangent to both ω1 and ω2 at
A and B, respectively. Let E, F be points on O1 O2 such that XE is tangent

to ω1 and Y F is tangent to ω2 . Let AE ∩ ω1 = A, C and BF ∩ ω2 = B, D.


Show that line BO2 is tangent to the circumcircle of △ACD and line AO2 is
tangent to △BCD.

Hide Solution

We show that both C and D lie on the circle with AB as diameter. Denote T
as the midpoint of AB (by power of point theorem T , X, Y are collinear).
From T A ⊥ AO1 and T B ⊥ BO2 this will entail that AO1 and BO2 tangent
to the circle with AB as diameter.
One way to see it is that by symmetry, we have Y E tangent to ω1 , too. Thus
E is the pole of XY w.r.t. ω1 . Also notice that the polar of A w.r.t. ω1 is AB

itself. Thus by La Hire's theorem, the pole of AE is the intersection of the


polar of A (i.e. AB) and the polar of E (i.e. XY ), which is T . With AE
intersecting ω1 at A and C , we know that T A and T C are both tangent to ω1
. Similarly, T B and T D are both tangent to ω2 . This entails
T A = T B = T C = T D, Q.E.D.

JOM 2013, G5. Let ABCD be a convex quadrilateral, with AD, BC


intersecting at F . Choose lines ℓ1 , ℓ2 such that ℓ1 passes through F and is
tangent to circle F AB, and ℓ2 passes through F and is tangent to circle F CD
. Let ℓ1 , ell2 intersect AC, BD at W , X, Y , Z . Prove that A, B, C, D are
concyclic if and only if W , X, Y , Z are concyclic.

Hide Solution

https://anzoteh96.github.io/jom_proposals.html 4/7
1/9/2019 JOM proposals

Suppose that ABCD is cyclic. Now ∠W F A = ∠F BA = ∠F DC = ∠ZF B ,


∠W AF = ∠DAC = ∠DBC = ∠ZBF . Considering triangles W AF and
ZBF , we know that ∠W F A = ∠ZF B, ∠W AF = ∠ZBF ,
∠AW F = ∠BZF ⇒ ∠XW Y = ∠XZY . So W XY Z is cyclic.

For the other direction we neeed the following lemma: Given a non-cyclic
convex quadrilateral ABCD, we have ∠CDA + ∠CBA > 180∘
⇔ ∠CBD > ∠CAD ⇔ ∠ADB > ∠ACB ⇔ ∠BDC > ∠BAC
⇔ ∠ABD > ∠ACD . Indeed, if we let C ′ be on AC with ABC ′ D cyclic,
then C lies outside the circle circumscribing ABC ′ D, and C ′ is on angle
domain DBC and angle domain ADC . Therefore,
∠CBD > ∠C BD = ∠CAD, ∠ADB = ∠AC B > ∠ACB,
′ ′

∠BDC > ∠BDC = ∠BAC , ∠ABD > ∠AC D = ∠ACD.


′ ′

Now assume that W XY Z is cyclic, but ABCD is not. W.l.o.g. let


. Now ∠ZF B = ∠F DC > ∠F BA = ∠W F A. Also

∠CDA + ∠CBA > 180
that ∠F W A = ∠F ZB, sine ∠XW Y = ∠XZY . So

∠CD = ∠W AF = 180 − ∠F W A = ∠W F A
> 180

. But from the lemma above
− ∠F ZB − ∠BF Z = ∠F BZ = ∠CBD
we have ∠CBD > ∠CAD, contradiction. The case where
∠CDA + ∠CBA < 180 is completely analogous.

JOM 2013, N5. Let p be an odd prime such that 2p + 1|pp + 1. Prove that
any prime divisor of 2 + 1 other than 3 is greater than 6p.
p

Hide Solution

Let q ≠ 3 be a prime dividing 2p + 1 and it is clear that q divides pp + 1. It is


also clear that 3|2p + 1 (since p is odd) and hence pp ≡ −1 (mod 3). So
p ≡ −1 (mod 3) ⇒ p ≡ −1 (mod 6). Also that
p

. Let r to be the minimal positive


p 2p
2 ≡ −1 (mod q) ⇒ 2 ≡ 1 (mod q)

integer such that 2 ≡ 1 (mod q). It is clear that r|2p, so r ∈ {1, 2, p, 2p}.
r

But r = 1, 2 implies that q|1 or q|3, which is impossible, and by (1), r ≠ p, so


r = 2p. But since ϕ(q) = q − 1 by Fermat's little theorem, 2p|q − 1, so

∃k ∈ N s.t. q = 2kp + 1.

p
q−1
We will now show that k ≠ 1, 2 . We have q|p
p
+ 1 ⇒ q|( ) + 1
2k
p p
(q−1) +(2k)
⇒ q|
(2k)
p
⇒ q| − 1 + (2k)
p
, since q is relatively prime to 2k . If k = 1 ,
from follows
, which is impossible. If k = 2, q = 4p + 1, and so
p
q|1 + 2 q|2

q = 4p + 1 ≡ 4(2) + 1 ≡ 0 (mod 3), contradicting the fact that q is prime.

Hence k ≥ 3 and q ≥ 6p + 1 > 6p, as desired.

JOM 2013, N6. Find all functions f : Z → Z such that for all prime p the
following condition holds:

p|ab + bc + ca ⇔ p|f (a)f (b) + f (b)f (c) + f (c)f (a).

Hide Solution

Answer: f (a) ≡ a or f (a) ≡ −a .

https://anzoteh96.github.io/jom_proposals.html 5/7
1/9/2019 JOM proposals

Denote ab + bc + ca as L and f (a)f (b) + f (b)f (c) + f (c)f (a) as R. It is not


hard to see that L = 0 ⇔ R = 0, since 0 is the only integer divisible by all
primes. Hence plugging a = b = c = 0 gives f (0) = 0. Also, |L| = 1 iff
|R| = 1, since 1 is the only integer divisible by none of the primes. Hence

plugging a = 0, b, c ∈ {1, −1} gives f (1), f (−1) ∈ {1, −1}. Letting


a = 0, b = 1 gives L = c, R = f (1)f (c), but since |f (1)| = 1 we have

|R| = |f (c)| so p|c iff p|f (c) for each prime p...(1).

Now suppose that for some integers m and n. Plugging


f (m) = f (n)

(a, b, c) = (m, −2m, −2m) gives L = 0, so

2f (m)f (−2m) + f (−2m)f (−2m) = R = 0. Plugging

(a, b, c) = (n, −2m, −2m) gives R = 2f (n)f (−2m) + f (−2m)f (−2m)

= 2f (m)f (−2m) + f (−2m)f (−2m) = 0. This forces L = 0, and therefore

4m
2
− 4mn = 0, or 4m(m − n) = 0. This forces m = n, or m = 0. The
latter case means that f (n) = f (0) = 0 for some integer n, but from (1) we
have p|n for all primes n. Hence n = m = 0, in this case. In either case we
have m = n, so f is injective...(2).

Now we prove that f is odd, i.e. f (−a) = −f (a) for all integers a . Assume
that f (a) + f (−a) ≠ 0 for some a. Let b = −a, then L = −a for all integers 2

c. This means p|a iff p|L iff p|R iff p|f (a)f (−a) + f (c)(f (a) + f (−a)). Now,

denote u = gcd(f (a) + f (−a), f (a)f (−a)), and for each integer x, denote
p(x) as the set of primes that divide x (so p(0) is the set of all primes, and

p(±1) is empty). We established the equivalence p(c) = p(f (c)) for all

integers c. Denote f (a) + f (−a) = xu and f (a)f (−a) = yu, so


gcd(x, y) = 1 and p(x) ⊆ p(a). Let's choose some c such that

p(c) = p(a)∖p(x), then since p(xy) = p(x) ∪ p(y), we have p(x) ∩ p(yf (c))

= p(x) ∩ (p(y) ∪ p(c)) = (p(x) ∩ p(y)) ∪ (p(x) ∩ p(c)) = ϕ ∪ ϕ = ϕ so x

and yf (c) are relatively prime. Recall that R = u(x + yf (c)). If


p(a)∖p(x) ≠ ϕ, then there exists infinitely c with p(c) = p(a)∖p(x) ≠ ϕ, so

we can choose such c satisfying x + yf (c) ≠ ±1 (the fact that f is injective


implies that there are infinitely many possible values of f (c)). Otherwise,
c = ±1 and with x ≠ 0 we know that either x + y or x − y is not in the set

{1, −1} (injectivity of f shows that f (1), f (−1) are precisely 1, −1). In either

case there is a prime q dividing x + yf (c), but with gcd(x, yf (c)) = 1, q


divides neither of them. This gives q ∉ p(x), p(y), p(c) and with
p(x) ∪ p(c) = p(a) we have q ∉ p(a), or q ∤ −a = L, contradiction.
2

Finally, we establish the equality f (kx) = kf (x) for all integers k, x. We


induct on k and the base case k = 1 is obvious. Now let f (kx) = kf (x) for
some k > 0 and for all x. Letting (a, b, c) = kx, −k(k + 1)x, −(k + 1)x gives
L = 0, so
0 = R = f (kx)f (−k(k + 1)x) + f (kx)f (−(k + 1)x) + f (−k(k + 1)x)f (−(k + 1)x)

= f (k(k + 1)x)f ((k + 1)x) − f (kx)f ((k + 1)x) − f (kx)f (k(k + 1)x)

(notice the implicit use of the identity f (−a) = −f (a)). The inductive
hypothesis f (kx) = kf (x) allows us to factor k out form the equation, so
0 = kf ((k + 1)x)f ((k + 1)x) − kf (x)f ((k + 1)x) − kf (x)kf ((k + 1)x)

= kf ((x + 1)x)(f ((k + 1)x) − f (x) − kf (x)) , since k > 0 anf

https://anzoteh96.github.io/jom_proposals.html 6/7
1/9/2019 JOM proposals

f ((x + 1)x) ≠ 0 for


, f ((k + 1)x) = (k + 1)f (x), as desired. In
x > 0

particular, f (c) = cf (1). If f (1) = 1, then f (c) = c for all integers c and if
f (1) = −1, then f (c) = −c for all integers c. Both functions satisfy the

problem condition since L = R in both cases. Q.E.D.

https://anzoteh96.github.io/jom_proposals.html 7/7

You might also like